Skip to Main Content

PrepTest 79, Game 2, Question 6

Transcript

Which one of the following is a permissible assignment of rangers to park areas. So this is your classic list question. We are looking for a possible arrangement of all the variables to the areas. Do we wanna remind ourselves about the implications of our master diagram? M has to be a 3, P and O can go on 1.

We have our conditional when O is in 2 and when O is in 3. And then of course we need to look out for either L or M or L or K. So the correct answer is E. We have JK and L in area 1. O and P in area 2 and M in area 3. So we're meeting our requirement that M goes in 3, everything looks good there.

Doesn't necessarily need a friend, it's okay that it's by itself. So here we have O in 2, so we know that J and K need to be in the same area. And we meet that requirement here, J and K occur in area 1. So neither P or O are occurring in 1, so that's good, and then L must be with M or K and we can see here that L is with K. Everything looks good here, this is our correct answer.

With the list question it's always good to take through those answer choices, the incorrect answer choices. Because sometimes if you've missed a rule or mischaracterized a rule it can really help you sort of work that out or work out even more global inferences to add to your diagram. So let's look at those.

Answer choice A, we have J K L O and then M P. Everything sort of looks good, we have O in 2 but then we have J and K together so that's good, M and P are fine here in 3 what we are missing is one of LM or LK. So we don't have that and this is the incorrect answer. For answer choice B we have KL OP and then JM. So we have O in 2 which means that we should expect to see J and K in the same area and we can see here that they are in different areas so this is not correct.

Answer choice C. We have K, and P, and 1, and we can cross this one out immediately because we know that P cannot go in 1. And then answer choice D, we have J, K and L. And then M and O. We can stop here because we know that M must go in 3.

So this is the incorrect answer.

Read full transcript